There are $n$ different $3$-element subsets $A_1,A_2,…,A_n$ of the set ${1,2,…,n}$, with $|A_i cap A_j|...












5












$begingroup$



Determine all possible values of positive integer $n$, such that there are $n$ different $3$-element subsets $A_1,A_2,...,A_n$ of the set ${1,2,...,n}$, with $|A_i cap A_j| not= 1$ for all $i not= j$.




Source: China Western Olympiad 2010





Attempt:



It is quite clear that for $n=4k$ such a system exist. For $n=4$, we have $A_1 ={1,2,3}$, $A_2 ={1,2,4}$, $A_3 ={2,3,4}$, $A_4 ={1,3,4}$. It is not hard to see that induction $nto n+4$ works. Now I would like to prove that there is no such system if $4nmid n$.



I thought about linear algebra approach. Observe the given sets as vectors in $mathbb{F}_2^n$. Then since $A_icdot A_i =1$ and $A_icdot A_j = 0$ for each $ine j$ these vectors are linear independent:
$$ b_1A_1+b_2A_2+...+b_nA_n = 0;;;; /cdot A_i$$
$$ b_1cdot 0+b_2cdot 0+...+b_icdot 1+...b_ncdot 0 =0implies b_i=0$$
But now, I'm not sure what to do...










share|cite|improve this question











$endgroup$

















    5












    $begingroup$



    Determine all possible values of positive integer $n$, such that there are $n$ different $3$-element subsets $A_1,A_2,...,A_n$ of the set ${1,2,...,n}$, with $|A_i cap A_j| not= 1$ for all $i not= j$.




    Source: China Western Olympiad 2010





    Attempt:



    It is quite clear that for $n=4k$ such a system exist. For $n=4$, we have $A_1 ={1,2,3}$, $A_2 ={1,2,4}$, $A_3 ={2,3,4}$, $A_4 ={1,3,4}$. It is not hard to see that induction $nto n+4$ works. Now I would like to prove that there is no such system if $4nmid n$.



    I thought about linear algebra approach. Observe the given sets as vectors in $mathbb{F}_2^n$. Then since $A_icdot A_i =1$ and $A_icdot A_j = 0$ for each $ine j$ these vectors are linear independent:
    $$ b_1A_1+b_2A_2+...+b_nA_n = 0;;;; /cdot A_i$$
    $$ b_1cdot 0+b_2cdot 0+...+b_icdot 1+...b_ncdot 0 =0implies b_i=0$$
    But now, I'm not sure what to do...










    share|cite|improve this question











    $endgroup$















      5












      5








      5





      $begingroup$



      Determine all possible values of positive integer $n$, such that there are $n$ different $3$-element subsets $A_1,A_2,...,A_n$ of the set ${1,2,...,n}$, with $|A_i cap A_j| not= 1$ for all $i not= j$.




      Source: China Western Olympiad 2010





      Attempt:



      It is quite clear that for $n=4k$ such a system exist. For $n=4$, we have $A_1 ={1,2,3}$, $A_2 ={1,2,4}$, $A_3 ={2,3,4}$, $A_4 ={1,3,4}$. It is not hard to see that induction $nto n+4$ works. Now I would like to prove that there is no such system if $4nmid n$.



      I thought about linear algebra approach. Observe the given sets as vectors in $mathbb{F}_2^n$. Then since $A_icdot A_i =1$ and $A_icdot A_j = 0$ for each $ine j$ these vectors are linear independent:
      $$ b_1A_1+b_2A_2+...+b_nA_n = 0;;;; /cdot A_i$$
      $$ b_1cdot 0+b_2cdot 0+...+b_icdot 1+...b_ncdot 0 =0implies b_i=0$$
      But now, I'm not sure what to do...










      share|cite|improve this question











      $endgroup$





      Determine all possible values of positive integer $n$, such that there are $n$ different $3$-element subsets $A_1,A_2,...,A_n$ of the set ${1,2,...,n}$, with $|A_i cap A_j| not= 1$ for all $i not= j$.




      Source: China Western Olympiad 2010





      Attempt:



      It is quite clear that for $n=4k$ such a system exist. For $n=4$, we have $A_1 ={1,2,3}$, $A_2 ={1,2,4}$, $A_3 ={2,3,4}$, $A_4 ={1,3,4}$. It is not hard to see that induction $nto n+4$ works. Now I would like to prove that there is no such system if $4nmid n$.



      I thought about linear algebra approach. Observe the given sets as vectors in $mathbb{F}_2^n$. Then since $A_icdot A_i =1$ and $A_icdot A_j = 0$ for each $ine j$ these vectors are linear independent:
      $$ b_1A_1+b_2A_2+...+b_nA_n = 0;;;; /cdot A_i$$
      $$ b_1cdot 0+b_2cdot 0+...+b_icdot 1+...b_ncdot 0 =0implies b_i=0$$
      But now, I'm not sure what to do...







      linear-algebra combinatorics contest-math algebraic-combinatorics






      share|cite|improve this question















      share|cite|improve this question













      share|cite|improve this question




      share|cite|improve this question








      edited Nov 26 '18 at 21:13







      greedoid

















      asked Jul 22 '18 at 18:53









      greedoidgreedoid

      39.5k114797




      39.5k114797






















          2 Answers
          2






          active

          oldest

          votes


















          2












          $begingroup$

          Suppose that there are $n$ such sets $A_1,A_2,ldots,A_n$, represented by indicator vectors $mathbf{a}_1,mathbf{a}_2,ldots,mathbf{a}_ninmathbb{F}_2^n$. Equip $mathbb{F}_2^n$ with the usual inner product $langle_,_rangle$.



          We already know that the vectors $mathbf{a}_1,mathbf{a}_2,ldots,mathbf{a}_n$ are linearly independent. Therefore, they span $mathbb{F}_2^n$. Thus, the vector $boldsymbol{1}:=(1,1,ldots,1)$ can be written as
          $$mathbf{a}_{j_1}+mathbf{a}_{j_2}+ldots+mathbf{a}_{j_k}$$
          for some $j_1,j_2,ldots,j_kin{1,2,ldots,n}=:[n]$ with $j_1<j_2<ldots<j_k$. If $k<n$, then there exists $rin[n]$ such that $rneq j_mu$ for all $mu=1,2,ldots,k$. That is,
          $$1=langle mathbf{a}_r,boldsymbol{1}rangle =sum_{mu=1}^k,langle mathbf{a}_{j_mu},mathbf{a}_rrangle=0,,$$
          which is a contradiction. Therefore, $k=n$, whence
          $$boldsymbol{1}=sum_{j=1}^n,mathbf{a}_j,.tag{*}$$
          Consequently, each element of $[n]$ belongs in an odd number of $A_1,A_2,ldots,A_n$, whence at least one of the sets $A_1,A_2,ldots,A_n$.



          Furthermore, it is not difficult to show that every element of $[n]$ must belong in at least two of the $A_i$'s. (If there exists an element of $[n]$ belonging in exactly one $A_j$, then you can show that there are at most $n-2$ possible $A_i$'s.) Let $d_j$ be the number of sets $A_i$ such that $jin A_i$. Then
          $$sum_{j=1}^n,d_j=3n,.tag{#}$$

          Note that $d_jgeq 2$ for all $jin[n]$.



          From (*), we conclude that $d_jgeq 3$ for every $jin[n]$. However, (#) implies that $d_j=3$ for all $jin[n]$; i.e., every element of $[n]$ must be in exactly three of the $A_i$'s. Write $mathbf{e}_1,mathbf{e}_2,ldots,mathbf{e}_n$ for the standard basis vectors of $mathbb{F}^n_2$. We see that
          $$mathbf{e}_j=mathbf{a}_p+mathbf{a}_q+mathbf{a}_r$$
          where $j$ is in $A_p$, $A_q$, and $A_r$. This shows that
          $$A_p={j,x,y},,,,A_q={j,y,z},,text{ and }A_r={j,z,x}$$
          for some $x,y,zin[n]$. Since $x$ already belongs in $A_p$ and $A_r$, it must be belong in another $A_s$. Clearly, $A_s$ must be equal to ${x,y,z}$. From here, we conclude that the four elements $j,x,y,z$ belong in exactly four of the $A_i$'s, which are ${j,x,y},{j,y,z},{j,z,x},{x,y,z}$. The rest is easy.






          share|cite|improve this answer











          $endgroup$













          • $begingroup$
            I read it all. All I have to check now why is $d_igeq 2$ for each $i$.
            $endgroup$
            – greedoid
            Jul 22 '18 at 19:55












          • $begingroup$
            If there is an element of $[n]$ contained in exactly in one of the $A_i$'s, say $1in{1,2,3}$, then split $A_2,A_3,ldots,A_n$ into two groups---those that contain ${2,3}$ and those that are disjoint from ${2,3}$. Now, if there are $k$ of those that contain ${2,3}$, then there are at most $n-k-3$ of those that are disjoint from ${2,3}$. Thus, you can end up with at most $1+k+(n-k-3)=n-2$ sets.
            $endgroup$
            – Batominovski
            Jul 22 '18 at 19:58












          • $begingroup$
            How you got $n-k-3$?
            $endgroup$
            – greedoid
            Jul 22 '18 at 20:07










          • $begingroup$
            The sets of the first kind must be of the form ${2,3,t_1},{2,3,t_2},ldots,{2,3,t_k}$, and the sets of the second kind must be disjoint from ${1,2,3,t_1,t_2,ldots,t_k}$. Therefore, the sets of the second kind are subsets of $[n]setminus {1,2,3,t_1,t_2,ldots,t_k}$, which has $n-k-3$ elements.
            $endgroup$
            – Batominovski
            Jul 22 '18 at 20:12












          • $begingroup$
            I still don't understand. Why does this mean that we have at most n-k-3 subsets?
            $endgroup$
            – greedoid
            Jul 22 '18 at 20:28



















          1












          $begingroup$


          Let $n$ be a positive integer. If $A_1,A_2,ldots,A_m$ are $3$-subsets of $[n]$ such that $left|A_icap A_jright|neq 1$ for $ineq j$, then the largest possible value of $m$ is
          $$m_max=left{
          begin{array}{ll}
          n&text{if }nequiv0pmod{4},,\
          n-1&text{if }nequiv1pmod{4},,\
          n-2&text{else},.
          end{array}
          right.$$




          Remark: Below is a sketch of my proof of the claim above. Be warned that a complete proof is quite long, whence I am providing a sketch with various gaps to be filled in. I hope that somebody will come up with a nicer proof.



          Proof. The first two cases follow from my first answer. I shall now deal with the last case, where $m_max=n-2$.



          Suppose contrary that there are $A_1,A_2,ldots,A_{n-1}$ satisfying the intersection condition. Then, proceed as before. The indicator vectors $mathbf{a}_1,mathbf{a}_2,ldots,mathbf{a}_{n-1}inmathbb{F}_2^n$ are linearly independent. Thus, there exists $mathbf{v}inmathbb{F}_2^n$ such that $mathbf{a}_1,mathbf{a}_2,ldots,mathbf{a}_{n-1},mathbf{b}$ form a basis of $mathbb{F}_2^n$. We can assume that $langle mathbf{a}_i,mathbf{b}rangle=0$ for all $i=1,2,ldots,n-1$ (otherwise, replace $mathbf{b}$ by $mathbf{b}-sum_{i=1}^{n-1},langle mathbf{a}_i,mathbf{b}rangle ,mathbf{a}_i$). Observe that $langle mathbf{b},mathbf{b}rangle=1$.



          Note that $$boldsymbol{1}=sum_{i=1}^{n-1},mathbf{a}_i+mathbf{b},.$$
          Let $B$ be the subset of $[n]$ with the indicator vector $mathbf{b}$. Let $X$ denote the set of $i$ such that $A_i$ is disjoint from $B$, and $Y$ the set of $i$ such that $A_icap B$ has two elements. Observe that $X$ and $Y$ form a partition of ${1,2,ldots,n-1}$; moreover,
          $$mathcal{X}:=bigcup_{iin X},A_itext{ and }mathcal{Y}:=bigcup_{iin Y},A_i$$
          are disjoint subsets of $[n]$.



          If $Xneq emptyset$, then we can use induction to finish the proof, noting that $A_isubseteq [n]setminus (Bcupmathcal{Y})$ for all $iin X$. From now on, assume that $X=emptyset$.



          Consider a simple graph $G$ on the vertex set $B$ where two vertices $i,jin B$ ($ineq j$) are connected by an edge iff $i$ and $j$ belongs in some $A_p$ simultaneously. If $C$ is a connected component of $G$ and $kin [n]setminus B$, then we say that $k$ is adjacent to $C$ if there exists $A_p$ such that $A_pcap B$ is an edge of $C$ and $kin A_p$, in which case, we also say that $A_p$ is incident to $C$. It is important to note that, if $C_1$ and $C_2$ are two distinct connected components of $G$, and $k_1,k_2in [n]setminus B$ are adjacent to $C_1$ and $C_2$, respectively, then $k_1neq k_2$.



          Let $C$ be a connected component of $G$ with at least two vertices. We have three probable scenarios:




          1. $C$ is a type-1 connected component, namely, $C$ is an isolated edge (i.e., it has only two vertices and one edge);

          2. $C$ is a type-2 connected component, namely, $C$ is a triangle (i.e., $C$ consists of $3$ vertices and $3$ edges);

          3. $C$ is a type-3 connected component, namely, $C$ is a star graph (i.e., there exists a vertex $v$ of $C$ such that every edge of $C$ takes the form ${v,w}$, where $w$ is any vertex of $C$ distinct from $v$).


          It can be readily seen that, if $C$ is a connected component of type 2 or type 3 of $G$, then $C$ is adjacent to exactly one element of $[n]setminus B$. If $G$ has a connected component $C$ of type 2, then the removal of vertices in $C$ along with the element $jin[n]setminus B$ which is adjacent to $C$ reduces the elements of $[n]$ by $4$, whilst ridding of only three sets $A_i$. Then, we finish the proof for this case by induction. Suppose from now on that $G$ has no connected components of type 2.



          Now, assume that $G$ has a connected component $C$ of type 3, which has $s$ vertices. Let $jin[n]setminus B$ be adjacent to $C$. Then, the removal of vertices of $C$ along with $j$ from $[n]$ reduces the elements of $[n]$ by $s+1$, whilst ridding of only $s-1$ sets $A_i$. Therefore, the claim hold trivially.



          Finally, assume that $G$ has only connected components of type 1 and possibly some isolated vertices. Then, it follows immediately that there are at most $n-2t$ sets $A_i$, where $t$ is the number of connected components of type 1. This shows that $t=0$. Thus, $G$ has only isolated vertices, but this is a contradiction as well (as $X=emptyset$ is assumed).






          share|cite|improve this answer











          $endgroup$













            Your Answer





            StackExchange.ifUsing("editor", function () {
            return StackExchange.using("mathjaxEditing", function () {
            StackExchange.MarkdownEditor.creationCallbacks.add(function (editor, postfix) {
            StackExchange.mathjaxEditing.prepareWmdForMathJax(editor, postfix, [["$", "$"], ["\\(","\\)"]]);
            });
            });
            }, "mathjax-editing");

            StackExchange.ready(function() {
            var channelOptions = {
            tags: "".split(" "),
            id: "69"
            };
            initTagRenderer("".split(" "), "".split(" "), channelOptions);

            StackExchange.using("externalEditor", function() {
            // Have to fire editor after snippets, if snippets enabled
            if (StackExchange.settings.snippets.snippetsEnabled) {
            StackExchange.using("snippets", function() {
            createEditor();
            });
            }
            else {
            createEditor();
            }
            });

            function createEditor() {
            StackExchange.prepareEditor({
            heartbeatType: 'answer',
            autoActivateHeartbeat: false,
            convertImagesToLinks: true,
            noModals: true,
            showLowRepImageUploadWarning: true,
            reputationToPostImages: 10,
            bindNavPrevention: true,
            postfix: "",
            imageUploader: {
            brandingHtml: "Powered by u003ca class="icon-imgur-white" href="https://imgur.com/"u003eu003c/au003e",
            contentPolicyHtml: "User contributions licensed under u003ca href="https://creativecommons.org/licenses/by-sa/3.0/"u003ecc by-sa 3.0 with attribution requiredu003c/au003e u003ca href="https://stackoverflow.com/legal/content-policy"u003e(content policy)u003c/au003e",
            allowUrls: true
            },
            noCode: true, onDemand: true,
            discardSelector: ".discard-answer"
            ,immediatelyShowMarkdownHelp:true
            });


            }
            });














            draft saved

            draft discarded


















            StackExchange.ready(
            function () {
            StackExchange.openid.initPostLogin('.new-post-login', 'https%3a%2f%2fmath.stackexchange.com%2fquestions%2f2859673%2fthere-are-n-different-3-element-subsets-a-1-a-2-a-n-of-the-set-1-2%23new-answer', 'question_page');
            }
            );

            Post as a guest















            Required, but never shown

























            2 Answers
            2






            active

            oldest

            votes








            2 Answers
            2






            active

            oldest

            votes









            active

            oldest

            votes






            active

            oldest

            votes









            2












            $begingroup$

            Suppose that there are $n$ such sets $A_1,A_2,ldots,A_n$, represented by indicator vectors $mathbf{a}_1,mathbf{a}_2,ldots,mathbf{a}_ninmathbb{F}_2^n$. Equip $mathbb{F}_2^n$ with the usual inner product $langle_,_rangle$.



            We already know that the vectors $mathbf{a}_1,mathbf{a}_2,ldots,mathbf{a}_n$ are linearly independent. Therefore, they span $mathbb{F}_2^n$. Thus, the vector $boldsymbol{1}:=(1,1,ldots,1)$ can be written as
            $$mathbf{a}_{j_1}+mathbf{a}_{j_2}+ldots+mathbf{a}_{j_k}$$
            for some $j_1,j_2,ldots,j_kin{1,2,ldots,n}=:[n]$ with $j_1<j_2<ldots<j_k$. If $k<n$, then there exists $rin[n]$ such that $rneq j_mu$ for all $mu=1,2,ldots,k$. That is,
            $$1=langle mathbf{a}_r,boldsymbol{1}rangle =sum_{mu=1}^k,langle mathbf{a}_{j_mu},mathbf{a}_rrangle=0,,$$
            which is a contradiction. Therefore, $k=n$, whence
            $$boldsymbol{1}=sum_{j=1}^n,mathbf{a}_j,.tag{*}$$
            Consequently, each element of $[n]$ belongs in an odd number of $A_1,A_2,ldots,A_n$, whence at least one of the sets $A_1,A_2,ldots,A_n$.



            Furthermore, it is not difficult to show that every element of $[n]$ must belong in at least two of the $A_i$'s. (If there exists an element of $[n]$ belonging in exactly one $A_j$, then you can show that there are at most $n-2$ possible $A_i$'s.) Let $d_j$ be the number of sets $A_i$ such that $jin A_i$. Then
            $$sum_{j=1}^n,d_j=3n,.tag{#}$$

            Note that $d_jgeq 2$ for all $jin[n]$.



            From (*), we conclude that $d_jgeq 3$ for every $jin[n]$. However, (#) implies that $d_j=3$ for all $jin[n]$; i.e., every element of $[n]$ must be in exactly three of the $A_i$'s. Write $mathbf{e}_1,mathbf{e}_2,ldots,mathbf{e}_n$ for the standard basis vectors of $mathbb{F}^n_2$. We see that
            $$mathbf{e}_j=mathbf{a}_p+mathbf{a}_q+mathbf{a}_r$$
            where $j$ is in $A_p$, $A_q$, and $A_r$. This shows that
            $$A_p={j,x,y},,,,A_q={j,y,z},,text{ and }A_r={j,z,x}$$
            for some $x,y,zin[n]$. Since $x$ already belongs in $A_p$ and $A_r$, it must be belong in another $A_s$. Clearly, $A_s$ must be equal to ${x,y,z}$. From here, we conclude that the four elements $j,x,y,z$ belong in exactly four of the $A_i$'s, which are ${j,x,y},{j,y,z},{j,z,x},{x,y,z}$. The rest is easy.






            share|cite|improve this answer











            $endgroup$













            • $begingroup$
              I read it all. All I have to check now why is $d_igeq 2$ for each $i$.
              $endgroup$
              – greedoid
              Jul 22 '18 at 19:55












            • $begingroup$
              If there is an element of $[n]$ contained in exactly in one of the $A_i$'s, say $1in{1,2,3}$, then split $A_2,A_3,ldots,A_n$ into two groups---those that contain ${2,3}$ and those that are disjoint from ${2,3}$. Now, if there are $k$ of those that contain ${2,3}$, then there are at most $n-k-3$ of those that are disjoint from ${2,3}$. Thus, you can end up with at most $1+k+(n-k-3)=n-2$ sets.
              $endgroup$
              – Batominovski
              Jul 22 '18 at 19:58












            • $begingroup$
              How you got $n-k-3$?
              $endgroup$
              – greedoid
              Jul 22 '18 at 20:07










            • $begingroup$
              The sets of the first kind must be of the form ${2,3,t_1},{2,3,t_2},ldots,{2,3,t_k}$, and the sets of the second kind must be disjoint from ${1,2,3,t_1,t_2,ldots,t_k}$. Therefore, the sets of the second kind are subsets of $[n]setminus {1,2,3,t_1,t_2,ldots,t_k}$, which has $n-k-3$ elements.
              $endgroup$
              – Batominovski
              Jul 22 '18 at 20:12












            • $begingroup$
              I still don't understand. Why does this mean that we have at most n-k-3 subsets?
              $endgroup$
              – greedoid
              Jul 22 '18 at 20:28
















            2












            $begingroup$

            Suppose that there are $n$ such sets $A_1,A_2,ldots,A_n$, represented by indicator vectors $mathbf{a}_1,mathbf{a}_2,ldots,mathbf{a}_ninmathbb{F}_2^n$. Equip $mathbb{F}_2^n$ with the usual inner product $langle_,_rangle$.



            We already know that the vectors $mathbf{a}_1,mathbf{a}_2,ldots,mathbf{a}_n$ are linearly independent. Therefore, they span $mathbb{F}_2^n$. Thus, the vector $boldsymbol{1}:=(1,1,ldots,1)$ can be written as
            $$mathbf{a}_{j_1}+mathbf{a}_{j_2}+ldots+mathbf{a}_{j_k}$$
            for some $j_1,j_2,ldots,j_kin{1,2,ldots,n}=:[n]$ with $j_1<j_2<ldots<j_k$. If $k<n$, then there exists $rin[n]$ such that $rneq j_mu$ for all $mu=1,2,ldots,k$. That is,
            $$1=langle mathbf{a}_r,boldsymbol{1}rangle =sum_{mu=1}^k,langle mathbf{a}_{j_mu},mathbf{a}_rrangle=0,,$$
            which is a contradiction. Therefore, $k=n$, whence
            $$boldsymbol{1}=sum_{j=1}^n,mathbf{a}_j,.tag{*}$$
            Consequently, each element of $[n]$ belongs in an odd number of $A_1,A_2,ldots,A_n$, whence at least one of the sets $A_1,A_2,ldots,A_n$.



            Furthermore, it is not difficult to show that every element of $[n]$ must belong in at least two of the $A_i$'s. (If there exists an element of $[n]$ belonging in exactly one $A_j$, then you can show that there are at most $n-2$ possible $A_i$'s.) Let $d_j$ be the number of sets $A_i$ such that $jin A_i$. Then
            $$sum_{j=1}^n,d_j=3n,.tag{#}$$

            Note that $d_jgeq 2$ for all $jin[n]$.



            From (*), we conclude that $d_jgeq 3$ for every $jin[n]$. However, (#) implies that $d_j=3$ for all $jin[n]$; i.e., every element of $[n]$ must be in exactly three of the $A_i$'s. Write $mathbf{e}_1,mathbf{e}_2,ldots,mathbf{e}_n$ for the standard basis vectors of $mathbb{F}^n_2$. We see that
            $$mathbf{e}_j=mathbf{a}_p+mathbf{a}_q+mathbf{a}_r$$
            where $j$ is in $A_p$, $A_q$, and $A_r$. This shows that
            $$A_p={j,x,y},,,,A_q={j,y,z},,text{ and }A_r={j,z,x}$$
            for some $x,y,zin[n]$. Since $x$ already belongs in $A_p$ and $A_r$, it must be belong in another $A_s$. Clearly, $A_s$ must be equal to ${x,y,z}$. From here, we conclude that the four elements $j,x,y,z$ belong in exactly four of the $A_i$'s, which are ${j,x,y},{j,y,z},{j,z,x},{x,y,z}$. The rest is easy.






            share|cite|improve this answer











            $endgroup$













            • $begingroup$
              I read it all. All I have to check now why is $d_igeq 2$ for each $i$.
              $endgroup$
              – greedoid
              Jul 22 '18 at 19:55












            • $begingroup$
              If there is an element of $[n]$ contained in exactly in one of the $A_i$'s, say $1in{1,2,3}$, then split $A_2,A_3,ldots,A_n$ into two groups---those that contain ${2,3}$ and those that are disjoint from ${2,3}$. Now, if there are $k$ of those that contain ${2,3}$, then there are at most $n-k-3$ of those that are disjoint from ${2,3}$. Thus, you can end up with at most $1+k+(n-k-3)=n-2$ sets.
              $endgroup$
              – Batominovski
              Jul 22 '18 at 19:58












            • $begingroup$
              How you got $n-k-3$?
              $endgroup$
              – greedoid
              Jul 22 '18 at 20:07










            • $begingroup$
              The sets of the first kind must be of the form ${2,3,t_1},{2,3,t_2},ldots,{2,3,t_k}$, and the sets of the second kind must be disjoint from ${1,2,3,t_1,t_2,ldots,t_k}$. Therefore, the sets of the second kind are subsets of $[n]setminus {1,2,3,t_1,t_2,ldots,t_k}$, which has $n-k-3$ elements.
              $endgroup$
              – Batominovski
              Jul 22 '18 at 20:12












            • $begingroup$
              I still don't understand. Why does this mean that we have at most n-k-3 subsets?
              $endgroup$
              – greedoid
              Jul 22 '18 at 20:28














            2












            2








            2





            $begingroup$

            Suppose that there are $n$ such sets $A_1,A_2,ldots,A_n$, represented by indicator vectors $mathbf{a}_1,mathbf{a}_2,ldots,mathbf{a}_ninmathbb{F}_2^n$. Equip $mathbb{F}_2^n$ with the usual inner product $langle_,_rangle$.



            We already know that the vectors $mathbf{a}_1,mathbf{a}_2,ldots,mathbf{a}_n$ are linearly independent. Therefore, they span $mathbb{F}_2^n$. Thus, the vector $boldsymbol{1}:=(1,1,ldots,1)$ can be written as
            $$mathbf{a}_{j_1}+mathbf{a}_{j_2}+ldots+mathbf{a}_{j_k}$$
            for some $j_1,j_2,ldots,j_kin{1,2,ldots,n}=:[n]$ with $j_1<j_2<ldots<j_k$. If $k<n$, then there exists $rin[n]$ such that $rneq j_mu$ for all $mu=1,2,ldots,k$. That is,
            $$1=langle mathbf{a}_r,boldsymbol{1}rangle =sum_{mu=1}^k,langle mathbf{a}_{j_mu},mathbf{a}_rrangle=0,,$$
            which is a contradiction. Therefore, $k=n$, whence
            $$boldsymbol{1}=sum_{j=1}^n,mathbf{a}_j,.tag{*}$$
            Consequently, each element of $[n]$ belongs in an odd number of $A_1,A_2,ldots,A_n$, whence at least one of the sets $A_1,A_2,ldots,A_n$.



            Furthermore, it is not difficult to show that every element of $[n]$ must belong in at least two of the $A_i$'s. (If there exists an element of $[n]$ belonging in exactly one $A_j$, then you can show that there are at most $n-2$ possible $A_i$'s.) Let $d_j$ be the number of sets $A_i$ such that $jin A_i$. Then
            $$sum_{j=1}^n,d_j=3n,.tag{#}$$

            Note that $d_jgeq 2$ for all $jin[n]$.



            From (*), we conclude that $d_jgeq 3$ for every $jin[n]$. However, (#) implies that $d_j=3$ for all $jin[n]$; i.e., every element of $[n]$ must be in exactly three of the $A_i$'s. Write $mathbf{e}_1,mathbf{e}_2,ldots,mathbf{e}_n$ for the standard basis vectors of $mathbb{F}^n_2$. We see that
            $$mathbf{e}_j=mathbf{a}_p+mathbf{a}_q+mathbf{a}_r$$
            where $j$ is in $A_p$, $A_q$, and $A_r$. This shows that
            $$A_p={j,x,y},,,,A_q={j,y,z},,text{ and }A_r={j,z,x}$$
            for some $x,y,zin[n]$. Since $x$ already belongs in $A_p$ and $A_r$, it must be belong in another $A_s$. Clearly, $A_s$ must be equal to ${x,y,z}$. From here, we conclude that the four elements $j,x,y,z$ belong in exactly four of the $A_i$'s, which are ${j,x,y},{j,y,z},{j,z,x},{x,y,z}$. The rest is easy.






            share|cite|improve this answer











            $endgroup$



            Suppose that there are $n$ such sets $A_1,A_2,ldots,A_n$, represented by indicator vectors $mathbf{a}_1,mathbf{a}_2,ldots,mathbf{a}_ninmathbb{F}_2^n$. Equip $mathbb{F}_2^n$ with the usual inner product $langle_,_rangle$.



            We already know that the vectors $mathbf{a}_1,mathbf{a}_2,ldots,mathbf{a}_n$ are linearly independent. Therefore, they span $mathbb{F}_2^n$. Thus, the vector $boldsymbol{1}:=(1,1,ldots,1)$ can be written as
            $$mathbf{a}_{j_1}+mathbf{a}_{j_2}+ldots+mathbf{a}_{j_k}$$
            for some $j_1,j_2,ldots,j_kin{1,2,ldots,n}=:[n]$ with $j_1<j_2<ldots<j_k$. If $k<n$, then there exists $rin[n]$ such that $rneq j_mu$ for all $mu=1,2,ldots,k$. That is,
            $$1=langle mathbf{a}_r,boldsymbol{1}rangle =sum_{mu=1}^k,langle mathbf{a}_{j_mu},mathbf{a}_rrangle=0,,$$
            which is a contradiction. Therefore, $k=n$, whence
            $$boldsymbol{1}=sum_{j=1}^n,mathbf{a}_j,.tag{*}$$
            Consequently, each element of $[n]$ belongs in an odd number of $A_1,A_2,ldots,A_n$, whence at least one of the sets $A_1,A_2,ldots,A_n$.



            Furthermore, it is not difficult to show that every element of $[n]$ must belong in at least two of the $A_i$'s. (If there exists an element of $[n]$ belonging in exactly one $A_j$, then you can show that there are at most $n-2$ possible $A_i$'s.) Let $d_j$ be the number of sets $A_i$ such that $jin A_i$. Then
            $$sum_{j=1}^n,d_j=3n,.tag{#}$$

            Note that $d_jgeq 2$ for all $jin[n]$.



            From (*), we conclude that $d_jgeq 3$ for every $jin[n]$. However, (#) implies that $d_j=3$ for all $jin[n]$; i.e., every element of $[n]$ must be in exactly three of the $A_i$'s. Write $mathbf{e}_1,mathbf{e}_2,ldots,mathbf{e}_n$ for the standard basis vectors of $mathbb{F}^n_2$. We see that
            $$mathbf{e}_j=mathbf{a}_p+mathbf{a}_q+mathbf{a}_r$$
            where $j$ is in $A_p$, $A_q$, and $A_r$. This shows that
            $$A_p={j,x,y},,,,A_q={j,y,z},,text{ and }A_r={j,z,x}$$
            for some $x,y,zin[n]$. Since $x$ already belongs in $A_p$ and $A_r$, it must be belong in another $A_s$. Clearly, $A_s$ must be equal to ${x,y,z}$. From here, we conclude that the four elements $j,x,y,z$ belong in exactly four of the $A_i$'s, which are ${j,x,y},{j,y,z},{j,z,x},{x,y,z}$. The rest is easy.







            share|cite|improve this answer














            share|cite|improve this answer



            share|cite|improve this answer








            edited Jul 23 '18 at 11:40

























            answered Jul 22 '18 at 19:21









            BatominovskiBatominovski

            1




            1












            • $begingroup$
              I read it all. All I have to check now why is $d_igeq 2$ for each $i$.
              $endgroup$
              – greedoid
              Jul 22 '18 at 19:55












            • $begingroup$
              If there is an element of $[n]$ contained in exactly in one of the $A_i$'s, say $1in{1,2,3}$, then split $A_2,A_3,ldots,A_n$ into two groups---those that contain ${2,3}$ and those that are disjoint from ${2,3}$. Now, if there are $k$ of those that contain ${2,3}$, then there are at most $n-k-3$ of those that are disjoint from ${2,3}$. Thus, you can end up with at most $1+k+(n-k-3)=n-2$ sets.
              $endgroup$
              – Batominovski
              Jul 22 '18 at 19:58












            • $begingroup$
              How you got $n-k-3$?
              $endgroup$
              – greedoid
              Jul 22 '18 at 20:07










            • $begingroup$
              The sets of the first kind must be of the form ${2,3,t_1},{2,3,t_2},ldots,{2,3,t_k}$, and the sets of the second kind must be disjoint from ${1,2,3,t_1,t_2,ldots,t_k}$. Therefore, the sets of the second kind are subsets of $[n]setminus {1,2,3,t_1,t_2,ldots,t_k}$, which has $n-k-3$ elements.
              $endgroup$
              – Batominovski
              Jul 22 '18 at 20:12












            • $begingroup$
              I still don't understand. Why does this mean that we have at most n-k-3 subsets?
              $endgroup$
              – greedoid
              Jul 22 '18 at 20:28


















            • $begingroup$
              I read it all. All I have to check now why is $d_igeq 2$ for each $i$.
              $endgroup$
              – greedoid
              Jul 22 '18 at 19:55












            • $begingroup$
              If there is an element of $[n]$ contained in exactly in one of the $A_i$'s, say $1in{1,2,3}$, then split $A_2,A_3,ldots,A_n$ into two groups---those that contain ${2,3}$ and those that are disjoint from ${2,3}$. Now, if there are $k$ of those that contain ${2,3}$, then there are at most $n-k-3$ of those that are disjoint from ${2,3}$. Thus, you can end up with at most $1+k+(n-k-3)=n-2$ sets.
              $endgroup$
              – Batominovski
              Jul 22 '18 at 19:58












            • $begingroup$
              How you got $n-k-3$?
              $endgroup$
              – greedoid
              Jul 22 '18 at 20:07










            • $begingroup$
              The sets of the first kind must be of the form ${2,3,t_1},{2,3,t_2},ldots,{2,3,t_k}$, and the sets of the second kind must be disjoint from ${1,2,3,t_1,t_2,ldots,t_k}$. Therefore, the sets of the second kind are subsets of $[n]setminus {1,2,3,t_1,t_2,ldots,t_k}$, which has $n-k-3$ elements.
              $endgroup$
              – Batominovski
              Jul 22 '18 at 20:12












            • $begingroup$
              I still don't understand. Why does this mean that we have at most n-k-3 subsets?
              $endgroup$
              – greedoid
              Jul 22 '18 at 20:28
















            $begingroup$
            I read it all. All I have to check now why is $d_igeq 2$ for each $i$.
            $endgroup$
            – greedoid
            Jul 22 '18 at 19:55






            $begingroup$
            I read it all. All I have to check now why is $d_igeq 2$ for each $i$.
            $endgroup$
            – greedoid
            Jul 22 '18 at 19:55














            $begingroup$
            If there is an element of $[n]$ contained in exactly in one of the $A_i$'s, say $1in{1,2,3}$, then split $A_2,A_3,ldots,A_n$ into two groups---those that contain ${2,3}$ and those that are disjoint from ${2,3}$. Now, if there are $k$ of those that contain ${2,3}$, then there are at most $n-k-3$ of those that are disjoint from ${2,3}$. Thus, you can end up with at most $1+k+(n-k-3)=n-2$ sets.
            $endgroup$
            – Batominovski
            Jul 22 '18 at 19:58






            $begingroup$
            If there is an element of $[n]$ contained in exactly in one of the $A_i$'s, say $1in{1,2,3}$, then split $A_2,A_3,ldots,A_n$ into two groups---those that contain ${2,3}$ and those that are disjoint from ${2,3}$. Now, if there are $k$ of those that contain ${2,3}$, then there are at most $n-k-3$ of those that are disjoint from ${2,3}$. Thus, you can end up with at most $1+k+(n-k-3)=n-2$ sets.
            $endgroup$
            – Batominovski
            Jul 22 '18 at 19:58














            $begingroup$
            How you got $n-k-3$?
            $endgroup$
            – greedoid
            Jul 22 '18 at 20:07




            $begingroup$
            How you got $n-k-3$?
            $endgroup$
            – greedoid
            Jul 22 '18 at 20:07












            $begingroup$
            The sets of the first kind must be of the form ${2,3,t_1},{2,3,t_2},ldots,{2,3,t_k}$, and the sets of the second kind must be disjoint from ${1,2,3,t_1,t_2,ldots,t_k}$. Therefore, the sets of the second kind are subsets of $[n]setminus {1,2,3,t_1,t_2,ldots,t_k}$, which has $n-k-3$ elements.
            $endgroup$
            – Batominovski
            Jul 22 '18 at 20:12






            $begingroup$
            The sets of the first kind must be of the form ${2,3,t_1},{2,3,t_2},ldots,{2,3,t_k}$, and the sets of the second kind must be disjoint from ${1,2,3,t_1,t_2,ldots,t_k}$. Therefore, the sets of the second kind are subsets of $[n]setminus {1,2,3,t_1,t_2,ldots,t_k}$, which has $n-k-3$ elements.
            $endgroup$
            – Batominovski
            Jul 22 '18 at 20:12














            $begingroup$
            I still don't understand. Why does this mean that we have at most n-k-3 subsets?
            $endgroup$
            – greedoid
            Jul 22 '18 at 20:28




            $begingroup$
            I still don't understand. Why does this mean that we have at most n-k-3 subsets?
            $endgroup$
            – greedoid
            Jul 22 '18 at 20:28











            1












            $begingroup$


            Let $n$ be a positive integer. If $A_1,A_2,ldots,A_m$ are $3$-subsets of $[n]$ such that $left|A_icap A_jright|neq 1$ for $ineq j$, then the largest possible value of $m$ is
            $$m_max=left{
            begin{array}{ll}
            n&text{if }nequiv0pmod{4},,\
            n-1&text{if }nequiv1pmod{4},,\
            n-2&text{else},.
            end{array}
            right.$$




            Remark: Below is a sketch of my proof of the claim above. Be warned that a complete proof is quite long, whence I am providing a sketch with various gaps to be filled in. I hope that somebody will come up with a nicer proof.



            Proof. The first two cases follow from my first answer. I shall now deal with the last case, where $m_max=n-2$.



            Suppose contrary that there are $A_1,A_2,ldots,A_{n-1}$ satisfying the intersection condition. Then, proceed as before. The indicator vectors $mathbf{a}_1,mathbf{a}_2,ldots,mathbf{a}_{n-1}inmathbb{F}_2^n$ are linearly independent. Thus, there exists $mathbf{v}inmathbb{F}_2^n$ such that $mathbf{a}_1,mathbf{a}_2,ldots,mathbf{a}_{n-1},mathbf{b}$ form a basis of $mathbb{F}_2^n$. We can assume that $langle mathbf{a}_i,mathbf{b}rangle=0$ for all $i=1,2,ldots,n-1$ (otherwise, replace $mathbf{b}$ by $mathbf{b}-sum_{i=1}^{n-1},langle mathbf{a}_i,mathbf{b}rangle ,mathbf{a}_i$). Observe that $langle mathbf{b},mathbf{b}rangle=1$.



            Note that $$boldsymbol{1}=sum_{i=1}^{n-1},mathbf{a}_i+mathbf{b},.$$
            Let $B$ be the subset of $[n]$ with the indicator vector $mathbf{b}$. Let $X$ denote the set of $i$ such that $A_i$ is disjoint from $B$, and $Y$ the set of $i$ such that $A_icap B$ has two elements. Observe that $X$ and $Y$ form a partition of ${1,2,ldots,n-1}$; moreover,
            $$mathcal{X}:=bigcup_{iin X},A_itext{ and }mathcal{Y}:=bigcup_{iin Y},A_i$$
            are disjoint subsets of $[n]$.



            If $Xneq emptyset$, then we can use induction to finish the proof, noting that $A_isubseteq [n]setminus (Bcupmathcal{Y})$ for all $iin X$. From now on, assume that $X=emptyset$.



            Consider a simple graph $G$ on the vertex set $B$ where two vertices $i,jin B$ ($ineq j$) are connected by an edge iff $i$ and $j$ belongs in some $A_p$ simultaneously. If $C$ is a connected component of $G$ and $kin [n]setminus B$, then we say that $k$ is adjacent to $C$ if there exists $A_p$ such that $A_pcap B$ is an edge of $C$ and $kin A_p$, in which case, we also say that $A_p$ is incident to $C$. It is important to note that, if $C_1$ and $C_2$ are two distinct connected components of $G$, and $k_1,k_2in [n]setminus B$ are adjacent to $C_1$ and $C_2$, respectively, then $k_1neq k_2$.



            Let $C$ be a connected component of $G$ with at least two vertices. We have three probable scenarios:




            1. $C$ is a type-1 connected component, namely, $C$ is an isolated edge (i.e., it has only two vertices and one edge);

            2. $C$ is a type-2 connected component, namely, $C$ is a triangle (i.e., $C$ consists of $3$ vertices and $3$ edges);

            3. $C$ is a type-3 connected component, namely, $C$ is a star graph (i.e., there exists a vertex $v$ of $C$ such that every edge of $C$ takes the form ${v,w}$, where $w$ is any vertex of $C$ distinct from $v$).


            It can be readily seen that, if $C$ is a connected component of type 2 or type 3 of $G$, then $C$ is adjacent to exactly one element of $[n]setminus B$. If $G$ has a connected component $C$ of type 2, then the removal of vertices in $C$ along with the element $jin[n]setminus B$ which is adjacent to $C$ reduces the elements of $[n]$ by $4$, whilst ridding of only three sets $A_i$. Then, we finish the proof for this case by induction. Suppose from now on that $G$ has no connected components of type 2.



            Now, assume that $G$ has a connected component $C$ of type 3, which has $s$ vertices. Let $jin[n]setminus B$ be adjacent to $C$. Then, the removal of vertices of $C$ along with $j$ from $[n]$ reduces the elements of $[n]$ by $s+1$, whilst ridding of only $s-1$ sets $A_i$. Therefore, the claim hold trivially.



            Finally, assume that $G$ has only connected components of type 1 and possibly some isolated vertices. Then, it follows immediately that there are at most $n-2t$ sets $A_i$, where $t$ is the number of connected components of type 1. This shows that $t=0$. Thus, $G$ has only isolated vertices, but this is a contradiction as well (as $X=emptyset$ is assumed).






            share|cite|improve this answer











            $endgroup$


















              1












              $begingroup$


              Let $n$ be a positive integer. If $A_1,A_2,ldots,A_m$ are $3$-subsets of $[n]$ such that $left|A_icap A_jright|neq 1$ for $ineq j$, then the largest possible value of $m$ is
              $$m_max=left{
              begin{array}{ll}
              n&text{if }nequiv0pmod{4},,\
              n-1&text{if }nequiv1pmod{4},,\
              n-2&text{else},.
              end{array}
              right.$$




              Remark: Below is a sketch of my proof of the claim above. Be warned that a complete proof is quite long, whence I am providing a sketch with various gaps to be filled in. I hope that somebody will come up with a nicer proof.



              Proof. The first two cases follow from my first answer. I shall now deal with the last case, where $m_max=n-2$.



              Suppose contrary that there are $A_1,A_2,ldots,A_{n-1}$ satisfying the intersection condition. Then, proceed as before. The indicator vectors $mathbf{a}_1,mathbf{a}_2,ldots,mathbf{a}_{n-1}inmathbb{F}_2^n$ are linearly independent. Thus, there exists $mathbf{v}inmathbb{F}_2^n$ such that $mathbf{a}_1,mathbf{a}_2,ldots,mathbf{a}_{n-1},mathbf{b}$ form a basis of $mathbb{F}_2^n$. We can assume that $langle mathbf{a}_i,mathbf{b}rangle=0$ for all $i=1,2,ldots,n-1$ (otherwise, replace $mathbf{b}$ by $mathbf{b}-sum_{i=1}^{n-1},langle mathbf{a}_i,mathbf{b}rangle ,mathbf{a}_i$). Observe that $langle mathbf{b},mathbf{b}rangle=1$.



              Note that $$boldsymbol{1}=sum_{i=1}^{n-1},mathbf{a}_i+mathbf{b},.$$
              Let $B$ be the subset of $[n]$ with the indicator vector $mathbf{b}$. Let $X$ denote the set of $i$ such that $A_i$ is disjoint from $B$, and $Y$ the set of $i$ such that $A_icap B$ has two elements. Observe that $X$ and $Y$ form a partition of ${1,2,ldots,n-1}$; moreover,
              $$mathcal{X}:=bigcup_{iin X},A_itext{ and }mathcal{Y}:=bigcup_{iin Y},A_i$$
              are disjoint subsets of $[n]$.



              If $Xneq emptyset$, then we can use induction to finish the proof, noting that $A_isubseteq [n]setminus (Bcupmathcal{Y})$ for all $iin X$. From now on, assume that $X=emptyset$.



              Consider a simple graph $G$ on the vertex set $B$ where two vertices $i,jin B$ ($ineq j$) are connected by an edge iff $i$ and $j$ belongs in some $A_p$ simultaneously. If $C$ is a connected component of $G$ and $kin [n]setminus B$, then we say that $k$ is adjacent to $C$ if there exists $A_p$ such that $A_pcap B$ is an edge of $C$ and $kin A_p$, in which case, we also say that $A_p$ is incident to $C$. It is important to note that, if $C_1$ and $C_2$ are two distinct connected components of $G$, and $k_1,k_2in [n]setminus B$ are adjacent to $C_1$ and $C_2$, respectively, then $k_1neq k_2$.



              Let $C$ be a connected component of $G$ with at least two vertices. We have three probable scenarios:




              1. $C$ is a type-1 connected component, namely, $C$ is an isolated edge (i.e., it has only two vertices and one edge);

              2. $C$ is a type-2 connected component, namely, $C$ is a triangle (i.e., $C$ consists of $3$ vertices and $3$ edges);

              3. $C$ is a type-3 connected component, namely, $C$ is a star graph (i.e., there exists a vertex $v$ of $C$ such that every edge of $C$ takes the form ${v,w}$, where $w$ is any vertex of $C$ distinct from $v$).


              It can be readily seen that, if $C$ is a connected component of type 2 or type 3 of $G$, then $C$ is adjacent to exactly one element of $[n]setminus B$. If $G$ has a connected component $C$ of type 2, then the removal of vertices in $C$ along with the element $jin[n]setminus B$ which is adjacent to $C$ reduces the elements of $[n]$ by $4$, whilst ridding of only three sets $A_i$. Then, we finish the proof for this case by induction. Suppose from now on that $G$ has no connected components of type 2.



              Now, assume that $G$ has a connected component $C$ of type 3, which has $s$ vertices. Let $jin[n]setminus B$ be adjacent to $C$. Then, the removal of vertices of $C$ along with $j$ from $[n]$ reduces the elements of $[n]$ by $s+1$, whilst ridding of only $s-1$ sets $A_i$. Therefore, the claim hold trivially.



              Finally, assume that $G$ has only connected components of type 1 and possibly some isolated vertices. Then, it follows immediately that there are at most $n-2t$ sets $A_i$, where $t$ is the number of connected components of type 1. This shows that $t=0$. Thus, $G$ has only isolated vertices, but this is a contradiction as well (as $X=emptyset$ is assumed).






              share|cite|improve this answer











              $endgroup$
















                1












                1








                1





                $begingroup$


                Let $n$ be a positive integer. If $A_1,A_2,ldots,A_m$ are $3$-subsets of $[n]$ such that $left|A_icap A_jright|neq 1$ for $ineq j$, then the largest possible value of $m$ is
                $$m_max=left{
                begin{array}{ll}
                n&text{if }nequiv0pmod{4},,\
                n-1&text{if }nequiv1pmod{4},,\
                n-2&text{else},.
                end{array}
                right.$$




                Remark: Below is a sketch of my proof of the claim above. Be warned that a complete proof is quite long, whence I am providing a sketch with various gaps to be filled in. I hope that somebody will come up with a nicer proof.



                Proof. The first two cases follow from my first answer. I shall now deal with the last case, where $m_max=n-2$.



                Suppose contrary that there are $A_1,A_2,ldots,A_{n-1}$ satisfying the intersection condition. Then, proceed as before. The indicator vectors $mathbf{a}_1,mathbf{a}_2,ldots,mathbf{a}_{n-1}inmathbb{F}_2^n$ are linearly independent. Thus, there exists $mathbf{v}inmathbb{F}_2^n$ such that $mathbf{a}_1,mathbf{a}_2,ldots,mathbf{a}_{n-1},mathbf{b}$ form a basis of $mathbb{F}_2^n$. We can assume that $langle mathbf{a}_i,mathbf{b}rangle=0$ for all $i=1,2,ldots,n-1$ (otherwise, replace $mathbf{b}$ by $mathbf{b}-sum_{i=1}^{n-1},langle mathbf{a}_i,mathbf{b}rangle ,mathbf{a}_i$). Observe that $langle mathbf{b},mathbf{b}rangle=1$.



                Note that $$boldsymbol{1}=sum_{i=1}^{n-1},mathbf{a}_i+mathbf{b},.$$
                Let $B$ be the subset of $[n]$ with the indicator vector $mathbf{b}$. Let $X$ denote the set of $i$ such that $A_i$ is disjoint from $B$, and $Y$ the set of $i$ such that $A_icap B$ has two elements. Observe that $X$ and $Y$ form a partition of ${1,2,ldots,n-1}$; moreover,
                $$mathcal{X}:=bigcup_{iin X},A_itext{ and }mathcal{Y}:=bigcup_{iin Y},A_i$$
                are disjoint subsets of $[n]$.



                If $Xneq emptyset$, then we can use induction to finish the proof, noting that $A_isubseteq [n]setminus (Bcupmathcal{Y})$ for all $iin X$. From now on, assume that $X=emptyset$.



                Consider a simple graph $G$ on the vertex set $B$ where two vertices $i,jin B$ ($ineq j$) are connected by an edge iff $i$ and $j$ belongs in some $A_p$ simultaneously. If $C$ is a connected component of $G$ and $kin [n]setminus B$, then we say that $k$ is adjacent to $C$ if there exists $A_p$ such that $A_pcap B$ is an edge of $C$ and $kin A_p$, in which case, we also say that $A_p$ is incident to $C$. It is important to note that, if $C_1$ and $C_2$ are two distinct connected components of $G$, and $k_1,k_2in [n]setminus B$ are adjacent to $C_1$ and $C_2$, respectively, then $k_1neq k_2$.



                Let $C$ be a connected component of $G$ with at least two vertices. We have three probable scenarios:




                1. $C$ is a type-1 connected component, namely, $C$ is an isolated edge (i.e., it has only two vertices and one edge);

                2. $C$ is a type-2 connected component, namely, $C$ is a triangle (i.e., $C$ consists of $3$ vertices and $3$ edges);

                3. $C$ is a type-3 connected component, namely, $C$ is a star graph (i.e., there exists a vertex $v$ of $C$ such that every edge of $C$ takes the form ${v,w}$, where $w$ is any vertex of $C$ distinct from $v$).


                It can be readily seen that, if $C$ is a connected component of type 2 or type 3 of $G$, then $C$ is adjacent to exactly one element of $[n]setminus B$. If $G$ has a connected component $C$ of type 2, then the removal of vertices in $C$ along with the element $jin[n]setminus B$ which is adjacent to $C$ reduces the elements of $[n]$ by $4$, whilst ridding of only three sets $A_i$. Then, we finish the proof for this case by induction. Suppose from now on that $G$ has no connected components of type 2.



                Now, assume that $G$ has a connected component $C$ of type 3, which has $s$ vertices. Let $jin[n]setminus B$ be adjacent to $C$. Then, the removal of vertices of $C$ along with $j$ from $[n]$ reduces the elements of $[n]$ by $s+1$, whilst ridding of only $s-1$ sets $A_i$. Therefore, the claim hold trivially.



                Finally, assume that $G$ has only connected components of type 1 and possibly some isolated vertices. Then, it follows immediately that there are at most $n-2t$ sets $A_i$, where $t$ is the number of connected components of type 1. This shows that $t=0$. Thus, $G$ has only isolated vertices, but this is a contradiction as well (as $X=emptyset$ is assumed).






                share|cite|improve this answer











                $endgroup$




                Let $n$ be a positive integer. If $A_1,A_2,ldots,A_m$ are $3$-subsets of $[n]$ such that $left|A_icap A_jright|neq 1$ for $ineq j$, then the largest possible value of $m$ is
                $$m_max=left{
                begin{array}{ll}
                n&text{if }nequiv0pmod{4},,\
                n-1&text{if }nequiv1pmod{4},,\
                n-2&text{else},.
                end{array}
                right.$$




                Remark: Below is a sketch of my proof of the claim above. Be warned that a complete proof is quite long, whence I am providing a sketch with various gaps to be filled in. I hope that somebody will come up with a nicer proof.



                Proof. The first two cases follow from my first answer. I shall now deal with the last case, where $m_max=n-2$.



                Suppose contrary that there are $A_1,A_2,ldots,A_{n-1}$ satisfying the intersection condition. Then, proceed as before. The indicator vectors $mathbf{a}_1,mathbf{a}_2,ldots,mathbf{a}_{n-1}inmathbb{F}_2^n$ are linearly independent. Thus, there exists $mathbf{v}inmathbb{F}_2^n$ such that $mathbf{a}_1,mathbf{a}_2,ldots,mathbf{a}_{n-1},mathbf{b}$ form a basis of $mathbb{F}_2^n$. We can assume that $langle mathbf{a}_i,mathbf{b}rangle=0$ for all $i=1,2,ldots,n-1$ (otherwise, replace $mathbf{b}$ by $mathbf{b}-sum_{i=1}^{n-1},langle mathbf{a}_i,mathbf{b}rangle ,mathbf{a}_i$). Observe that $langle mathbf{b},mathbf{b}rangle=1$.



                Note that $$boldsymbol{1}=sum_{i=1}^{n-1},mathbf{a}_i+mathbf{b},.$$
                Let $B$ be the subset of $[n]$ with the indicator vector $mathbf{b}$. Let $X$ denote the set of $i$ such that $A_i$ is disjoint from $B$, and $Y$ the set of $i$ such that $A_icap B$ has two elements. Observe that $X$ and $Y$ form a partition of ${1,2,ldots,n-1}$; moreover,
                $$mathcal{X}:=bigcup_{iin X},A_itext{ and }mathcal{Y}:=bigcup_{iin Y},A_i$$
                are disjoint subsets of $[n]$.



                If $Xneq emptyset$, then we can use induction to finish the proof, noting that $A_isubseteq [n]setminus (Bcupmathcal{Y})$ for all $iin X$. From now on, assume that $X=emptyset$.



                Consider a simple graph $G$ on the vertex set $B$ where two vertices $i,jin B$ ($ineq j$) are connected by an edge iff $i$ and $j$ belongs in some $A_p$ simultaneously. If $C$ is a connected component of $G$ and $kin [n]setminus B$, then we say that $k$ is adjacent to $C$ if there exists $A_p$ such that $A_pcap B$ is an edge of $C$ and $kin A_p$, in which case, we also say that $A_p$ is incident to $C$. It is important to note that, if $C_1$ and $C_2$ are two distinct connected components of $G$, and $k_1,k_2in [n]setminus B$ are adjacent to $C_1$ and $C_2$, respectively, then $k_1neq k_2$.



                Let $C$ be a connected component of $G$ with at least two vertices. We have three probable scenarios:




                1. $C$ is a type-1 connected component, namely, $C$ is an isolated edge (i.e., it has only two vertices and one edge);

                2. $C$ is a type-2 connected component, namely, $C$ is a triangle (i.e., $C$ consists of $3$ vertices and $3$ edges);

                3. $C$ is a type-3 connected component, namely, $C$ is a star graph (i.e., there exists a vertex $v$ of $C$ such that every edge of $C$ takes the form ${v,w}$, where $w$ is any vertex of $C$ distinct from $v$).


                It can be readily seen that, if $C$ is a connected component of type 2 or type 3 of $G$, then $C$ is adjacent to exactly one element of $[n]setminus B$. If $G$ has a connected component $C$ of type 2, then the removal of vertices in $C$ along with the element $jin[n]setminus B$ which is adjacent to $C$ reduces the elements of $[n]$ by $4$, whilst ridding of only three sets $A_i$. Then, we finish the proof for this case by induction. Suppose from now on that $G$ has no connected components of type 2.



                Now, assume that $G$ has a connected component $C$ of type 3, which has $s$ vertices. Let $jin[n]setminus B$ be adjacent to $C$. Then, the removal of vertices of $C$ along with $j$ from $[n]$ reduces the elements of $[n]$ by $s+1$, whilst ridding of only $s-1$ sets $A_i$. Therefore, the claim hold trivially.



                Finally, assume that $G$ has only connected components of type 1 and possibly some isolated vertices. Then, it follows immediately that there are at most $n-2t$ sets $A_i$, where $t$ is the number of connected components of type 1. This shows that $t=0$. Thus, $G$ has only isolated vertices, but this is a contradiction as well (as $X=emptyset$ is assumed).







                share|cite|improve this answer














                share|cite|improve this answer



                share|cite|improve this answer








                edited Jul 23 '18 at 14:49

























                answered Jul 23 '18 at 13:13









                BatominovskiBatominovski

                1




                1






























                    draft saved

                    draft discarded




















































                    Thanks for contributing an answer to Mathematics Stack Exchange!


                    • Please be sure to answer the question. Provide details and share your research!

                    But avoid



                    • Asking for help, clarification, or responding to other answers.

                    • Making statements based on opinion; back them up with references or personal experience.


                    Use MathJax to format equations. MathJax reference.


                    To learn more, see our tips on writing great answers.




                    draft saved


                    draft discarded














                    StackExchange.ready(
                    function () {
                    StackExchange.openid.initPostLogin('.new-post-login', 'https%3a%2f%2fmath.stackexchange.com%2fquestions%2f2859673%2fthere-are-n-different-3-element-subsets-a-1-a-2-a-n-of-the-set-1-2%23new-answer', 'question_page');
                    }
                    );

                    Post as a guest















                    Required, but never shown





















































                    Required, but never shown














                    Required, but never shown












                    Required, but never shown







                    Required, but never shown

































                    Required, but never shown














                    Required, but never shown












                    Required, but never shown







                    Required, but never shown







                    Popular posts from this blog

                    Biblatex bibliography style without URLs when DOI exists (in Overleaf with Zotero bibliography)

                    ComboBox Display Member on multiple fields

                    Is it possible to collect Nectar points via Trainline?